Difference between revisions of "2025 AIME II Problems/Problem 4"

(Solution)
(Solution 2)
 
(12 intermediate revisions by 6 users not shown)
Line 6: Line 6:
 
We can rewrite the equation as:
 
We can rewrite the equation as:
  
= 15/12 * 24/21 * 35/32 * ... * 3968/3965 * \log_4 5 / \log_64 5
+
\begin{align*}
= \log_4 64 * (4+1)(4-1)(5+1)(5-1)* ... * (63+1)(63-1)/(4+2)(4-2)(5+2)(5-2)* ... * (63+2)(63-2)
+
&= \dfrac{15}{12} \cdot \dfrac{24}{21} \cdot \dfrac{35}{32} \cdot \dots \cdot \dfrac{3968}{3965} \cdot \dfrac{\log_4{5}}{\log_{64}{5}} \\
= 3 * 5 * 3 * 6 * 4 * ... * 64 * 62 / 6 * 2 * 7 * 3 * ... * 65 * 61
+
&= \log_4{64} \cdot \dfrac{(4+1)(4-1)(5+1)(5-1)\cdot \dots \cdot (63+1)(63-1)}{(4+2)(4-2)(5+2)(5-2)\cdot \dots \cdot (63+2)(63-2)} \\
= 3 * 5 * 62 / 65 * 2
+
&= 3 \cdot \dfrac{5 \cdot 3 \cdot 6 \cdot 4 \cdot \dots \cdot 64 \cdot 62}{6 \cdot 2 \cdot 7 \cdot 3 \cdot \dots \cdot 65 \cdot 61} \\
= 3 * 5 * 2 * 31 / 5 * 13 * 2
+
&= 3 \cdot \dfrac{5 \cdot 62}{65 \cdot 2} \\
= 3 * 31 / 13
+
&= 3 \cdot \dfrac{5 \cdot 2 \cdot 31}{5 \cdot 13 \cdot 2} \\
= 93/13
+
&= 3 \cdot \dfrac{31}{13} \\
Desired answer: 93 + 13 = 106
+
&= \dfrac{93}{13}
 +
\end{align*}
  
(Feel free to correct any latexes and formattings)
+
 
~Mitsuihisashi14
+
Desired answer: <math>93 + 13 = \boxed{106}</math>
 +
 
 +
(Feel free to correct any <math>\LaTeX</math> and formatting.)
 +
 
 +
~ Mitsuihisashi14
 +
 
 +
~ <math>\LaTeX</math> by [https://artofproblemsolving.com/wiki/index.php/User:Tacos_are_yummy_1 Tacos_are_yummy_1]
 +
 
 +
~ Additional edits by [https://artofproblemsolving.com/wiki/index.php/User:Aoum aoum]
 +
 
 +
== Solution 2==
 +
 
 +
We can move the exponents to the front of the logarithms like this:
 +
\begin{align*}
 +
\frac{\log_4 (5^{15})}{\log_5 (5^{12})} \cdot \frac{\log_5 (5^{24})}{\log_6 (5^{21})}\cdot \frac{\log_6 (5^{35})}{\log_7 (5^{32})} \cdots = \frac{15\log_4 (5)}{12\log_5 (5)} \cdot \frac{24\log_5 (5)}{21\log_6 (5)}\cdot \frac{35\log_6 (5)}{32\log_7 (5)} \cdots
 +
\end{align*}
 +
Now we multiply the logs and fractions separately. Let's do it for the logs first:
 +
\begin{align*}
 +
\frac{\log_4 (5)}{\log_5 (5)} \cdot \frac{\log_5 (5)}{\log_6 (5)}\cdot \frac{\log_6 (5)}{\log_7 (5)} \cdots \frac{\log_{63} (5)}{\log_{64} (5)} = \frac{\log_4 (5)}{\log_{64} (5)} = 3
 +
\end{align*}
 +
Now fractions:
 +
\begin{align*}
 +
\frac{15}{12} \cdot \frac{24}{21} \cdot \frac{35}{32} \cdots = \frac{3\cdot 5}{2\cdot 6} \cdot \frac{4\cdot 6}{3\cdot 7} \cdot \frac{5\cdot 7}{4\cdot 8} \cdots \frac{62\cdot 64}{61\cdot 65} = \frac{5}{2} \cdot \frac{62}{65} = \frac{31}{13}
 +
\end{align*}
 +
Multiplying these together gets us the original product, which is <math>\frac{31}{13} \cdot 3 = \frac{93}{13}</math>.
 +
Thus <math>m+n=\boxed{106}</math>.
 +
 
 +
~ Edited by [https://artofproblemsolving.com/wiki/index.php/User:Aoum aoum]
 +
 
 +
== Solution 3 ==
 +
 
 +
Using logarithmic identities and the change of base formula, the product can be rewritten as <cmath>\prod_{k=4}^{63}\frac{k^2-1}{k^2-4}\frac{\log(k+1)}{\log(k)}</cmath>. Then we can separate this into two series.
 +
The latter series is a telescoping series, and it can be pretty easily evaluated to be <math>\frac{\log(64)}{\log(4)}=3</math>. The former can be factored as <math>\frac{(k-1)(k+1)}{(k-2)(k+2)}</math>, and writing out the first terms could tell us that this is a telescoping series as well. Cancelling out the terms would yield <math>\frac{5}{2}\cdot\frac{62}{65}=\frac{31}{13}</math>.
 +
Multiplying the two will give us <math>\frac{93}{13}</math>, which tells us that the answer is <math>\boxed{106}</math>.
 +
 
 +
==Solution 4 (thorough)==
 +
 
 +
The product is equal to <math>\prod^{63}_{k=4} \frac{(k-1)(k+1)\log_k 5}{(k-2)(k+2)\log_{k + 1} 5}</math> from difference of squares and properties of logarithms. We can now expand:
 +
 
 +
\begin{align*}
 +
\prod^{63}_{k=4} \frac{(k-1)(k+1)\log_k 5}{(k-2)(k+2)\log_{k + 1} 5}
 +
&= \prod^{63}_{k=4}\frac{\log_k 5}{\log_{k+1} 5} \cdot \frac{3 \cdot 5 \cdot 4 \cdot 6 \cdot 5 \cdot 7 \cdots 62 \cdot 64}{2 \cdot 6 \cdot 3 \cdot 7 \cdot 4 \cdot 8 \cdots 61 \cdot 65} \\
 +
&= \frac{\log_4 5 \cdot \log_5 5 \cdots \log_{63} 5}{\log_{5} 5 \cdot \log_6 5 \cdots \log_{64} 5} \cdot \frac{3 \cdot 4 \cdot (5^2 \cdot 6^2 \cdots 62^2) \cdot 63 \cdot 64}{2 \cdot 3 \cdot 4 \cdot 5 \cdot (6^2 \cdot 7^2 \cdots 61^2) \cdot 62 \cdot 63 \cdot 64 \cdot 65} \\
 +
&= \frac{\log_4 5}{\log_{64} 5} \cdot \frac{3 \cdot 4 \cdot 5^2 \cdot (6^2 \cdots 61^2) \cdot 62^2 \cdot 63 \cdot 64}{2 \cdot 3 \cdot 4 \cdot 5 \cdot (6^2 \cdots 61^2) \cdot 62 \cdot 63 \cdot 64 \cdot 65} \\
 +
&= \log_{64} 4 \cdot \frac{3 \cdot 4 \cdot 5 \cdot 62}{2 \cdot 65} \\
 +
&= 3 \cdot \frac{31}{13} \\
 +
&= \frac{93}{13}
 +
\end{align*}
 +
 
 +
Thus the answer is <math>93+13=\boxed{106}</math>.
 +
 
 +
~ [https://artofproblemsolving.com/wiki/index.php/User:Eevee9406 eevee9406]
 +
 
 +
~ Edited by [https://artofproblemsolving.com/wiki/index.php/User:Aoum aoum]
 +
 
 +
==See also==
 +
{{AIME box|year=2025|num-b=3|num-a=5|n=II}}
 +
 
 +
{{MAA Notice}}

Latest revision as of 20:03, 15 February 2025

Problem

The product\[\prod^{63}_{k=4} \frac{\log_k (5^{k^2 - 1})}{\log_{k + 1} (5^{k^2 - 4})} = \frac{\log_4 (5^{15})}{\log_5 (5^{12})} \cdot \frac{\log_5 (5^{24})}{\log_6 (5^{21})}\cdot \frac{\log_6 (5^{35})}{\log_7 (5^{32})} \cdots \frac{\log_{63} (5^{3968})}{\log_{64} (5^{3965})}\]is equal to $\tfrac mn,$ where $m$ and $n$ are relatively prime positive integers. Find $m + n.$

Solution 1

We can rewrite the equation as:

\begin{align*} &= \dfrac{15}{12} \cdot \dfrac{24}{21} \cdot \dfrac{35}{32} \cdot \dots \cdot \dfrac{3968}{3965} \cdot \dfrac{\log_4{5}}{\log_{64}{5}} \\ &= \log_4{64} \cdot \dfrac{(4+1)(4-1)(5+1)(5-1)\cdot \dots \cdot (63+1)(63-1)}{(4+2)(4-2)(5+2)(5-2)\cdot \dots \cdot (63+2)(63-2)} \\ &= 3 \cdot \dfrac{5 \cdot 3 \cdot 6 \cdot 4 \cdot \dots \cdot 64 \cdot 62}{6 \cdot 2 \cdot 7 \cdot 3 \cdot \dots \cdot 65 \cdot 61} \\ &= 3 \cdot \dfrac{5 \cdot 62}{65 \cdot 2} \\ &= 3 \cdot \dfrac{5 \cdot 2 \cdot 31}{5 \cdot 13 \cdot 2} \\ &= 3 \cdot \dfrac{31}{13} \\ &= \dfrac{93}{13} \end{align*}


Desired answer: $93 + 13 = \boxed{106}$

(Feel free to correct any $\LaTeX$ and formatting.)

~ Mitsuihisashi14

~ $\LaTeX$ by Tacos_are_yummy_1

~ Additional edits by aoum

Solution 2

We can move the exponents to the front of the logarithms like this: \begin{align*} \frac{\log_4 (5^{15})}{\log_5 (5^{12})} \cdot \frac{\log_5 (5^{24})}{\log_6 (5^{21})}\cdot \frac{\log_6 (5^{35})}{\log_7 (5^{32})} \cdots = \frac{15\log_4 (5)}{12\log_5 (5)} \cdot \frac{24\log_5 (5)}{21\log_6 (5)}\cdot \frac{35\log_6 (5)}{32\log_7 (5)} \cdots \end{align*} Now we multiply the logs and fractions separately. Let's do it for the logs first: \begin{align*} \frac{\log_4 (5)}{\log_5 (5)} \cdot \frac{\log_5 (5)}{\log_6 (5)}\cdot \frac{\log_6 (5)}{\log_7 (5)} \cdots \frac{\log_{63} (5)}{\log_{64} (5)} = \frac{\log_4 (5)}{\log_{64} (5)} = 3 \end{align*} Now fractions: \begin{align*} \frac{15}{12} \cdot \frac{24}{21} \cdot \frac{35}{32} \cdots = \frac{3\cdot 5}{2\cdot 6} \cdot \frac{4\cdot 6}{3\cdot 7} \cdot \frac{5\cdot 7}{4\cdot 8} \cdots \frac{62\cdot 64}{61\cdot 65} = \frac{5}{2} \cdot \frac{62}{65} = \frac{31}{13} \end{align*} Multiplying these together gets us the original product, which is $\frac{31}{13} \cdot 3 = \frac{93}{13}$. Thus $m+n=\boxed{106}$.

~ Edited by aoum

Solution 3

Using logarithmic identities and the change of base formula, the product can be rewritten as \[\prod_{k=4}^{63}\frac{k^2-1}{k^2-4}\frac{\log(k+1)}{\log(k)}\]. Then we can separate this into two series. The latter series is a telescoping series, and it can be pretty easily evaluated to be $\frac{\log(64)}{\log(4)}=3$. The former can be factored as $\frac{(k-1)(k+1)}{(k-2)(k+2)}$, and writing out the first terms could tell us that this is a telescoping series as well. Cancelling out the terms would yield $\frac{5}{2}\cdot\frac{62}{65}=\frac{31}{13}$. Multiplying the two will give us $\frac{93}{13}$, which tells us that the answer is $\boxed{106}$.

Solution 4 (thorough)

The product is equal to $\prod^{63}_{k=4} \frac{(k-1)(k+1)\log_k 5}{(k-2)(k+2)\log_{k + 1} 5}$ from difference of squares and properties of logarithms. We can now expand:

\begin{align*} \prod^{63}_{k=4} \frac{(k-1)(k+1)\log_k 5}{(k-2)(k+2)\log_{k + 1} 5} &= \prod^{63}_{k=4}\frac{\log_k 5}{\log_{k+1} 5} \cdot \frac{3 \cdot 5 \cdot 4 \cdot 6 \cdot 5 \cdot 7 \cdots 62 \cdot 64}{2 \cdot 6 \cdot 3 \cdot 7 \cdot 4 \cdot 8 \cdots 61 \cdot 65} \\ &= \frac{\log_4 5 \cdot \log_5 5 \cdots \log_{63} 5}{\log_{5} 5 \cdot \log_6 5 \cdots \log_{64} 5} \cdot \frac{3 \cdot 4 \cdot (5^2 \cdot 6^2 \cdots 62^2) \cdot 63 \cdot 64}{2 \cdot 3 \cdot 4 \cdot 5 \cdot (6^2 \cdot 7^2 \cdots 61^2) \cdot 62 \cdot 63 \cdot 64 \cdot 65} \\ &= \frac{\log_4 5}{\log_{64} 5} \cdot \frac{3 \cdot 4 \cdot 5^2 \cdot (6^2 \cdots 61^2) \cdot 62^2 \cdot 63 \cdot 64}{2 \cdot 3 \cdot 4 \cdot 5 \cdot (6^2 \cdots 61^2) \cdot 62 \cdot 63 \cdot 64 \cdot 65} \\ &= \log_{64} 4 \cdot \frac{3 \cdot 4 \cdot 5 \cdot 62}{2 \cdot 65} \\ &= 3 \cdot \frac{31}{13} \\ &= \frac{93}{13} \end{align*}

Thus the answer is $93+13=\boxed{106}$.

~ eevee9406

~ Edited by aoum

See also

2025 AIME II (ProblemsAnswer KeyResources)
Preceded by
Problem 3
Followed by
Problem 5
1 2 3 4 5 6 7 8 9 10 11 12 13 14 15
All AIME Problems and Solutions

The problems on this page are copyrighted by the Mathematical Association of America's American Mathematics Competitions. AMC logo.png